LSAT and Law School Admissions Forum

Get expert LSAT preparation and law school admissions advice from PowerScore Test Preparation.

User avatar
 Dave Killoran
PowerScore Staff
  • PowerScore Staff
  • Posts: 5853
  • Joined: Mar 25, 2011
|
#44135
Complete Question Explanation
(The complete setup for this game can be found here: lsat/viewtopic.php?t=16592)

The correct answer choice is (B)

Currently there is one associate, O. O must be promoted to partner this year, and at a minimum one assistant must be promoted, who in a sense “replaces” O. The same situation applies next year, where to minimize the number of associates the one associate would be promoted, but then be replaced by the minimum of one assistant that must be promoted. Thus, the correct answer is one, which is answer choice (B).
 gracekim
  • Posts: 2
  • Joined: Sep 28, 2012
|
#22259
I couldn't figure out 24. If anyone can help me out it would be greatly appreciated. Thank you!
 Jon Denning
PowerScore Staff
  • PowerScore Staff
  • Posts: 904
  • Joined: Apr 11, 2011
|
#22260
Hey Grace (Kim?) - thanks for the question. It would help me out if you could be a little more specific about exactly what gave you trouble with the question, but I'll try to add some insight anyway.

Since J is never voted for by either O or H, then J cannot be promoted the first year (cannot get the majority of O, R, and H above her) and must remain an assistant. The rest of the assistants can all potentially become associates during this year's review. At next year's review all of the associates could become partners, and J could finally be promoted to assistant. So that would be one assistant (J) minimum, answer choice B.

Hope that helps!
 gracekim
  • Posts: 2
  • Joined: Sep 28, 2012
|
#22261
Thank you for your reply. I reread the problem and I missed a rule! Doh!
 MeliXi
  • Posts: 19
  • Joined: Dec 12, 2020
|
#82224
Jon Denning wrote:Hey Grace (Kim?) - thanks for the question. It would help me out if you could be a little more specific about exactly what gave you trouble with the question, but I'll try to add some insight anyway.

Since J is never voted for by either O or H, then J cannot be promoted the first year (cannot get the majority of O, R, and H above her) and must remain an assistant. The rest of the assistants can all potentially become associates during this year's review. At next year's review all of the associates could become partners, and J could finally be promoted to assistant. So that would be one assistant (J) minimum, answer choice B.

Hope that helps!

Is it just me or is this explanation... not right?? Seems to be answering smallest possible number of assistants, rather than associates??
 Robert Carroll
PowerScore Staff
  • PowerScore Staff
  • Posts: 1787
  • Joined: Dec 06, 2013
|
#83479
Mel,

Everything before the last sentence of Jon's post seems to be focused entirely on how to get all the original assistants (besides J) promoted to associate in the first year, so that, after the next year's review, all those people could be promoted to partner. Everyone but J would thus not be an associate after next year's review, which is doing our best to minimize associates, like the question asks. Because we can't promote J to associate this year, but can't avoid promoting an assistant to associate every year until there are no assistants left, J will be an assistant after this year, and then will have to get promoted to associate after next year's review. If we could have promoted J quicker, then NO associates would remain after next year's review, which would have been the best we could do, except that J's inability to get promoted right away ruined that. So the best we can do is to have J alone as an associate after next year's review.

The logic of that is sound! I think there's just a typo in the last sentence (and it looks like the last word of the next-to-last sentence). If we change the last two instances of "assistant" to "associate", the explanation needs no substantive changes, and should validate Jon's point. If there's another error with the explanation, I'm not seeing it!

Robert Carroll

Get the most out of your LSAT Prep Plus subscription.

Analyze and track your performance with our Testing and Analytics Package.